LSAT and Law School Admissions Forum

Get expert LSAT preparation and law school admissions advice from PowerScore Test Preparation.

 LSAT2018
  • Posts: 242
  • Joined: Jan 10, 2018
|
#44779
How does this stimulus and answer ('The presence of the Apo-A-IV-2 gene may inhibit the elevation of blood cholesterol') compare with Flaw in the Reasoning questions with correlation/cause errors? I know this is a Must Be True question but I would like to clear things up.
The stimulus indicates, 'A high level of cholesterol in the blood is associated with an increased risk of heart' (correlation) and the answer seems to be causal. How is this valid?
 Adam Tyson
PowerScore Staff
  • PowerScore Staff
  • Posts: 5153
  • Joined: Apr 14, 2011
|
#44796
What makes it an okay answer for a "most strongly supported" question, LSAT2018, is two things: first, the presence of "may" in the answer choice means that the answer doesn't commit us to making a causal error. If two things are correlated, it's acceptable to say that one of them may have caused the other, even though it would be a flaw to say that one of them did cause the other.

The second thing is the question type, which is Most Strongly Supported. We treat this type as a subset of Must Be True questions, but it is a weaker variant because the stimulus needs only to support the answer, not prove it. In other words, the correct answer doesn't have to be true, but only has to be the one that gets the most support from the stimulus. None of the other answers gets any support from the stimulus, so answer E, which gets some, is the one that gets the most, right? Think of it as the difference between a Strengthen question and a Justify the Conclusion question, but in this case it is the stimulus that is strengthening the answer rather than vice versa.

Beware that subtle distinction between a full-power Must Be True question and its weaker variant, Most Strongly Supported. While some MSS answers will also be things that MBT, they don't have to be.
 LSAT2018
  • Posts: 242
  • Joined: Jan 10, 2018
|
#44931
Would the opposite of answer (D) be acceptable as an answer then? So if the answer says the presence of the Apo-A-IV-1 gene seems to indicate that a person has a higher risk of heart disease, this would be acceptable?

Otherwise, I`m going to take it that the part on the correlation ('A high level of cholesterol in the blood is associated with an increased risk of heart disease') is just a distraction in the stimulus that is more concerned with what caused the difference in cholesterol levels?
 Malila Robinson
PowerScore Staff
  • PowerScore Staff
  • Posts: 296
  • Joined: Feb 01, 2018
|
#45064
Hi LSAT2018,
No, this was linked to diet. So Type 1 might indicate a higher (or not lower) risk of heart disease *if* the person was eating a diet that was high in cholesterol. The diet is what is adding cholesterol to the blood and that seems to raise the risk of heart disease for Type 1 folks. D doesn't mention anything about the diet, and that info would be needed for the opposite answer to work.
Hope that helps,
Malila
 yupyup
  • Posts: 10
  • Joined: Jul 27, 2019
|
#66869
Even though I ultimately chose the correct answer, I was momentarily tempted by answer choice C. Could someone please explain why that isn't correct?
User avatar
 Dave Killoran
PowerScore Staff
  • PowerScore Staff
  • Posts: 5853
  • Joined: Mar 25, 2011
|
#66879
yupyup wrote:Even though I ultimately chose the correct answer, I was momentarily tempted by answer choice C. Could someone please explain why that isn't correct?
Hi Yup,

Take a look at how specific answer choice (C) is: do we really have any evidence that their bodies excreted cholesterol? There's no discussion of that at all, and so it would be very hard for this answer to be supported by what is said in the stimulus. All we know is that after eating these foods there were different levels, but we can't say that excretion caused those different levels.

This answer would be right for something like a Resolve question, but not in a Must/Most Strongly supported question :-D

Thanks!
 yupyup
  • Posts: 10
  • Joined: Jul 27, 2019
|
#66941
Dave Killoran wrote:
yupyup wrote:Even though I ultimately chose the correct answer, I was momentarily tempted by answer choice C. Could someone please explain why that isn't correct?
Hi Yup,

Take a look at how specific answer choice (C) is: do we really have any evidence that their bodies excreted cholesterol? There's no discussion of that at all, and so it would be very hard for this answer to be supported by what is said in the stimulus. All we know is that after eating these foods there were different levels, but we can't say that excretion caused those different levels.

This answer would be right for something like a Resolve question, but not in a Must/Most Strongly supported question :-D

Thanks!
Ah! That makes sense. I have to use the info in the stimulus to prove one of the answer choices, not the other way around, as I was apparently trying to do. Thanks so much Dave! :)

Get the most out of your LSAT Prep Plus subscription.

Analyze and track your performance with our Testing and Analytics Package.